Kehrtwende im Deep Space

Beim Schreiben eines physikalisch realistischen Spiels ("Asteroid Defender") kam eine physikalische Frage auf, ob Diag.1 oder Diag.2 oder Diag.3 die Realität korrekt abbilden.

Im Weltraum (abseits von anderen Himmelskörpern) mbewegt sich ein perfekt kugelförmiger Asteroid mit Masse in einer geraden Linie mit Geschwindigkeit v 0 relativ zum Punkt C(roter Punkt). Seine Bewegung ist konstant und gleichförmig, da keine Kräfte auf ihn einwirken.

Der Asteroid hat eine einheitliche Dichte, sodass sein Massenmittelpunkt (CoM) mit seinem geometrischen Zentrum zusammenfällt. Der Asteroid ist starr und verformt sich nicht, wenn er berührt oder geschoben wird. Der Asteroid dreht sich NICHT um sein CoM. Die blassgrünen Rechtecke, die auf dem Asteroiden erscheinen, visualisieren das Fehlen des Spins des Asteroiden. Dies ist zeitweise t-1und t0in den Diagrammen dargestellt.

Zeitweise t1wendet ein manövrierfähiger Raumschlepper (Weltraumschieber für europäische Leser) eine Kraft an F 1 an der Oberfläche des Asteroiden P1(kleiner gelber Punkt) über eine starre und flache Druckplatte, die vor dem Raumschlepper (dicke blaue Linie) montiert ist. Dieser Kraftvektor liegt auf einer Linie, die den Punkt „P1“ und das CoM verbindet, und kann daher den Asteroiden nicht dazu bringen, sich um sein CoM zu drehen.

Mit fortschreitender Zeit variiert der Raumschlepper kontinuierlichr die Richtung der ausgeübten Kraft in einer solchen Weise, dass der Asteroid eine halbkreisförmige Bahn (U-Turn) mit einem Radius um Punkt zentriert durchläuft C. Die Größe dieser Kraft bleibt während der gesamten Kehrtwende konstant – nur ihre Richtung ändert sich kontinuierlich.
Die angewandten Kraftvektoren liegen zu jeder Zeit auf Linien, die das CoM mit den Punkten verbinden, an denen die Druckplatte die Oberfläche des Asteroiden berührt (z. B.: P1bei t1, P2bei t2, P3bei t3, P4bei t4, P5beit5). Die Druckplatte rutscht NICHT auf der Oberfläche des Asteroiden und dreht ihn nicht um sein CoM - die Druckplatte drückt nur den Asteroiden. Dies ist in den Diagrammen zu Zeiten von t1bis dargestellt t5.

Sobald der Asteroid die 180-Grad-Wende vollzogen hat, löst sich der Raumschlepper und erlaubt dem Asteroiden, sich in einer geraden Linie mit der Geschwindigkeit zu entfernen v 0 was parallel, aber entgegengesetzt zum ursprünglichen Ansatz ist. Die kinetische Energie des Asteroiden vor und nach der Kehrtwende ist gleich. Der Asteroid dreht sich nicht um sein CoM, wenn er abfliegt. Dies ist manchmal t6und t7in den Diagrammen dargestellt.

FRAGE : Welches Diagramm bildet die Realität in diesem Szenario korrekt ab?

Bitte begründen Sie, warum ein Diagramm die Realität korrekt darstellt und die anderen nicht.

Diag. 1, stellt die Linien ( P1_CoM, ... P5_CoM) dar, die das CoM des Asteroiden und die Punkte verbinden, an denen die Druckplatte die Oberfläche des Asteroiden berührt ( P1at t1, ... P5at t5), wie sie immer durch die Mitte der Kehrtwendung verlaufen ( Punkt C). Die Vektoren ( F 1 , ... F 5 ) liegen auf diesen Linien. Zoomen Sie für weitere Details. Geben Sie hier die Bildbeschreibung einDiag. 2 und Abb. 3 zeigen die Linien ( P1_CoM, ... P5_CoM), die das CoM des Asteroiden und die Punkte verbinden, an denen die Druckplatte die Oberfläche des Asteroiden berührt ( P1at t1, ... P5at t5), als durch Punkte ( Q1, ... Q5) verlaufend, die NICHT mit dem Punkt zusammenfallen C.
Mit anderen Worten: die Linien ( P1_Q1, ... P5_Q5), auf denen die Kraftvektoren liegen ( F 1 , ... F 5 ), in einem bestimmten Abstand xvom Punkt entfernt passieren C.
Zoomen Sie für weitere Details. Geben Sie hier die Bildbeschreibung einZoomen Sie für weitere Details.Geben Sie hier die Bildbeschreibung ein

Die rot gestrichelte Linie P0_Q0ist nur eine Hilfslinie, die durch das CoM bei t1und durch das CoM bei t5und durch den Punkt verläuft C. Diese Linie ist ohne Vergrößern nicht zu sehen.

-------------- EDIT ----------------
In den Kommentaren zu Kamils ​​Antwort tauchte eine Frage auf, ob es möglich ist, eine Summe von zu haben zwei Vektoren A + B so dass die Größe dieser Summe gleich der Größe des Vektors ist A allein?
Die Antwort ist "Ja", aber das ist nur möglich, wenn der Winkel zwischen diesen beiden Vektoren >90º und <270º ist. Den formalen Beweis finden Sie hier: https://imgur.com/LELihq9

Eine weitere BEARBEITUNG: Als Antwort auf den Einwand von Luke Pritchett in den Kommentaren unten verlinke ich eine Antwort, die für seinen Einwand relevant ist: Asteroid Spin Prevention while Pushing

Gibt es keinen Stapel über Spiele und Spieledesign?
Ja, das gibt es, aber das ist eine physikalische Frage. Die Anwendung bestimmt nicht ihre Art.
"Die Druckplatte rutscht NICHT auf der Oberfläche des Asteroiden und dreht ihn nicht um sein CoM - die Druckplatte drückt nur den Asteroiden." Ich bin mir ziemlich sicher, dass das widersprüchlich ist. Damit der Schub den Asteroiden nicht in Rotation versetzt, muss die Kraft immer durch CoM zu Punkt c zeigen. Das heißt, der Angriffspunkt muss sich auch auf der Linie von CoM zu Punkt c drehen, der sich dreht. Dazu muss entweder die Platte an der Außenseite des Asteroiden entlang gleiten oder der Asteroid muss sich mit der gleichen Geschwindigkeit drehen, mit der er sich um c dreht.
@Luke: Es gibt eine dritte Lösung. Die Druckplatte kann relativ auf der Oberfläche des Asteroiden "rollen" ... und genau das zeigen die Diagramme. Außerdem kann ich Ihnen nicht zustimmen, dass: "... Damit die Druckplatte den Asteroiden nicht dreht, muss die Kraft immer durch CoM zu Punkt C zeigen" - das Zeigen auf den Punkt ist nicht erforderlich, um ein Drehen zu verhindern, sondern nur Czeigen to CoM ist zur Spin-Prävention notwendig. Ich kann eine andere Frage dazu eröffnen, wenn es hilft.
Die Kraft muss auf c zeigen, damit sich der Asteroid mit konstanter Geschwindigkeit auf einer Kreisbahn um c bewegt. Die Kraft muss auf das CoM zeigen, damit sich der Planet nicht dreht. Daher muss die Kraft durch beide zeigen. Ans Rollen habe ich allerdings nicht gedacht. Damit ist Diagramm 1 korrekt.
Hier also keine Reibung und damit die Kontaktkraft immer senkrecht zum Paddel?
ja72: Ja, der Kraftvektor steht immer senkrecht auf der Druckplatte und liegt immer auf der Linie Px_Qx, die durch das CoM des Asteroiden geht. Mit anderen Worten: Der Kraftvektor „zeigt“ immer auf den CoM.
Als Nebenbemerkung denke ich, dass ein Spiel wie dieses eine Menge Spaß machen würde. Der Versuch, die Flugbahn eines massiven Asteroiden nur durch eine Kontaktkraft (mit Reibung) zu kontrollieren.

Antworten (4)

Um eine halbkreisförmige Flugbahn zu erhalten, muss die Querbeschleunigung ungleich Null und konstant sein. Es ist ganz einfach. Wenn sich der Asteroid mit Geschwindigkeit bewegt v , und eine konstante Querbeschleunigung von A = A T angewendet wird, dann geht der Asteroid eine Kurve mit einem Krümmungsradius gleich zu R = v 2 / A T . Die Sweep-Rate wird sein ω = A T / v . Die Austrittsgeschwindigkeit ist v , da es keine Längsbeschleunigung gibt, um den Asteroiden zu beschleunigen oder zu verlangsamen.

Dies entspricht Diagramm 1.

Die Diagramme 2 und 3 sind falsch, da der Asteroid keine halbkreisförmige Bahn durchqueren wird. Beide sind Teilmengen des allgemeinen Problems, bei dem die Wirkungslinie einen Momentarm hat D vom momentanen Rotationszentrum (Punkt C ). Für Diagramm 2, D > 0 und für Diagramm 3 D < 0 . Natürlich ist Diagramm 1 D = 0 .

Unter Berücksichtigung des Steigungswinkels θ geformt von D über R (der Krümmungsradius) die Beschleunigung A wird in zwei Komponenten zerlegt

(1) A T = A cos θ A L = A Sünde θ

Die Trigonometrie des Problems ist so, dass D = R Sünde θ

skizzieren

Die Bewegungsgleichungen lauten:

(2) v ˙ = A Sünde θ v 2 R = A cos θ

Die Lösung des Obigen ist zu jedem Zeitpunkt

R = D 2 + ( v 2 A ) 2 v ˙ = A 2 D v 4 + A 2 D 2

Das heißt, der Radius hängt von der Geschwindigkeit ab, und die Geschwindigkeit ändert sich je nach Vorzeichen nichtlinear D . Somit ändert sich die Bahnkrümmung mit der Zeit und macht den Asteroiden spiralförmig .

Wie bringt man die Addition einer Geschwindigkeit v_a(verursacht durch die Beschleunigung a) senkrecht zur Anfangsgeschwindigkeit v( vor dieser Addition) in Einklang, die zu einer Vektorsumme führt v_sum, die nach diesem Beweis einen Betrag größer als die Anfangsgeschwindigkeit haben muss v?
@GeorgeRobinson - es ist bekannt, dass eine senkrechte Beschleunigung nur die Richtung der Geschwindigkeit und nicht ihre Größe ändert.
Warum? Ändert eine Beschleunigung über ein beliebiges Zeitintervall ungleich Null nicht immer die Größe der Geschwindigkeitskomponente, die auf derselben Linie liegt wie der Beschleunigungsvektor (oder Kraftvektor)? ...wie in diesem Beispiel
@GeorgeRobinson Es gibt niemals eine Tangentenkomponente der Beschleunigung und daher ändert sich die Geschwindigkeit nicht. Die Kommentare sind kein guter Ort, um dies zu beweisen. Stellen Sie eine neue Frage, wenn Sie mir nicht glauben, speziell zur Querbeschleunigung.
Ich möchte eine neue Frage zur senkrechten Beschleunigung erstellen. Könnten Sie den Titel dieser Frage vorschlagen, damit ich Ihre Worte nicht verdrehe?
Diese Frage wurde bereits beantwortet , so dass keine neue Frage erforderlich ist.

Zu jedem Zeitpunkt ändert die Kraftkomponente in der Linie (tangential zu) der momentanen Geschwindigkeit die Größe der Geschwindigkeit (dh Geschwindigkeit), aber nicht die Richtung; die Kraftkomponente senkrecht (normal) zur Linie der momentanen Geschwindigkeit ändert die Richtung der Geschwindigkeit, aber nicht ihre Größe.

In Diag. 1 steht die Kraft immer senkrecht auf der Geraden der momentanen Geschwindigkeit, also bleibt die Geschwindigkeit erhalten v 0 .

In Diag. 2 es gibt immer eine Kraftkomponente gegen die Geschwindigkeit; dies reduziert die Geschwindigkeit, also kann es nicht sein v 0 am Ende des Manövers.

In Diag. 3 gibt es immer eine Kraftkomponente, die zur Geschwindigkeit hinzukommt, also kann es nicht sein v 0 am Ende des Manövers.

In beiden Fällen kann sich der Asteroid entlang des Halbkreises bewegen, aber 2 und 3 erfordern, dass der Raumschlepper allmählich die Größe der senkrechten Komponente der Kraft ändert, nicht nur die Richtung. Dies liegt daran, dass die senkrechte Komponente eine Masse halten würde M auf der gegebenen Kreisbahn mit dem Radius R hängt von der Geschwindigkeit ab v :

F P = M v 2 R

Ich denke, es ist möglich, die Größe der Kraft in den Fällen 2 und 3 konstant zu halten. Eine nicht konstante senkrechte Komponente würde eine nicht konstante Tangentenkomponente erfordern, sodass die Gesamtgröße konstant bleiben könnte. Dennoch würde die Tangentenkomponente ungleich Null die Geschwindigkeit über die Zeit verringern (Diag. 2) oder erhöhen (Diag. 3).

Von den drei Diagrammen kann Ihnen nur das erste etwas geben v 0 .


Beachten Sie, dass eine Kehrtwende im Weltraum eine Verschwendung von Treibstoff ist. Wenn der Raumschlepper nur eine Kraft nach links ausübte, könnte er den Asteroiden schließlich stoppen und ihn dann beschleunigen v 0 . Flugzeuge in der Atmosphäre machen Kehrtwendungen entlang Halbkreisen, weil es sehr einfach ist, Normalkräfte aus der Aerodynamik zu erhalten; Außerdem müssen sie die Geschwindigkeit beibehalten, damit sie nicht ins Stocken geraten. Wenn Sie im Weltraum keine bestimmte Flugbahn benötigen, drücken Sie einfach lange genug nach links, um sich zu ändern v 0 Zu v 0 .

@GeorgeRobinson Wenn beide Vektoren senkrecht zur Geschwindigkeit sind, liegen sie auf einer Ebene senkrecht zur Geschwindigkeit, also liegt ihre Summe auf derselben Ebene, also ist die Summe auch senkrecht zur Geschwindigkeit.
Welche Vektoren meinst du? Wenn wir über vec-Addition sprechen, sprechen wir über das Addieren von Geschwindigkeitsvektoren zu Geschwindigkeitsvektoren oder von Kraftvektoren zu Kraftvektoren. Es ist offensichtlich, dass Kraft Beschleunigung verursacht (a=F/m), aber Beschleunigung ist nicht dasselbe wie Geschwindigkeit ... obwohl es die Änderungsrate davon ist. Sie schrieben: "... ändert die Richtung der Geschwindigkeit, aber nicht ihre Größe." - warum nicht auch die Größenordnung? Wenn ich einen Stein horizontal werfe, verursacht die senkrechte Gravitationsanziehung eine parabelförmige Flugbahn, in der sich sowohl die Größe als auch die Richtung der Geschwindigkeit des Steins ändern.
@GeorgeRobinson, wo ist also die Schwerkraft, die Sie für den Stein erwähnen - Sie geben an, dass Sie sich im Weltraum befinden, damit sich der Stein in einer geraden Linie bewegt.
@Mike: Das Felsenbeispiel war nur eine Veranschaulichung, dass eine Addition eines anderen senkrechten Vektors die Richtung und Größe der Vektorsumme ändert. In meinem Original-Posting gibt es keine Schwerkraft. Das andere, wenn auch verwandte parabolische Szenario kann jedoch mit einem Triebwerk anstelle der Schwerkraft erreicht werden (und sich wie ein horizontaler Steinwurf verhalten). Siehe: imgur.com/9dwtEPu
ANMERKUNG: Wie in diesem Beispiel das Auftreten von Beschleunigung in Y-Richtung dazu führt, dass die Geschwindigkeiten V1, ... V8 zur Senkrechten V0 addiert werden, was SOWOHL die Größe als auch die Richtung der resultierenden Vektorsumme VT ändert. Alles ohne Schwerkraft. Siehe: imgur.com/9dwtEPu
@GeorgeRobinson In diesem Beispiel ist die Kraft (immer in Y-Richtung) nicht immer senkrecht zur momentanen Geschwindigkeit. Wenn es nicht senkrecht ist, existiert eine Tangentenkomponente.
Aber in der ERSTEN INSTANZ IST der addierte Vektor senkrecht zu V0, jedoch ist SOWOHL der Betrag der Summe größer als der Betrag von V0 UND die Richtung der Summe unterscheidet sich von der Richtung von V0. In Ihrer Antwort schreiben Sie jedoch, dass der senkrechte Summand " ... die Richtung der Geschwindigkeit ändert, aber nicht ihre Größe ", und das stimmt einfach nicht, wie dieses Beispiel zeigt. Ja, es kann vorkommen, dass das Ergebnis der Vektoraddition eine andere Richtung, aber keinen Betrag hat, aber nur unter den besonderen Bedingungen, die in diesem Beweis erwähnt werden: imgur.com/LELihq9
@GeorgeRobinson Sie müssen in Begriffen denken D v , nicht Δ v . Stellen Sie sich stattdessen einen konstanten Schub und eine volle Parabel vor. Die Geschwindigkeit (Betrag der Geschwindigkeit) hat ihr Minimum am Scheitelpunkt ("top") der Parabel. Minimum, und es ändert sich sanft, sodass seine Ableitung in diesem Moment genau Null ist. Dies ist auch der einzige Moment, in dem die Kraft (Schub) senkrecht zur momentanen Geschwindigkeit steht. Es ist kein Zufall. In diesem Moment ändert die Kraft nicht die Geschwindigkeit, sie ändert nur die Richtung der Geschwindigkeit.
@Kamil: An der Spitze dieser Parabel entspricht die Gesamtgeschwindigkeit der horizontalen Geschwindigkeit, da die vertikale Geschwindigkeit Null ist. Im nächsten Fall wird die vertikale Geschwindigkeit größer als Null, somit nimmt die Größe der Gesamtgeschwindigkeit zu UND ihre Richtung ändert sich (während die horizontale Geschwindigkeit unbeeinflusst bleibt) .
@GeorgeRobinson Unter einer der anderen Antworten lautet der Kommentar "Sie müssen unendlich kleine Zeitänderungen berücksichtigen". Mehr könnte ich nicht zustimmen. Wenn Sie im Fall unserer vollen Parabel damit Probleme haben, kann es hilfreich sein, wenn Sie den Moment kurz vor dem Gipfel und den Moment kurz danach betrachten, sodass der Gipfel genau dazwischen liegt. In diesen beiden Momenten ist der Betrag der Geschwindigkeit gleich, die Richtung verschieden. Die Geschwindigkeit nimmt vor dem Gipfel ab, steigt nach dem Gipfel an. Die Spitze ist genau der Punkt, an dem die Geschwindigkeit weder abnimmt noch zunimmt.
@Kamil: Unendliche Zeitänderung ist gleichbedeutend mit unendlich kleinem Zeitintervall. Infinitesmal ist unendlich klein, aber immer noch größer als Null. Das Zeitintervall kann nicht Null sein, denn ohne Zeit gibt es keine Bewegung. Bewegung zu einem Zeitpunkt ist nicht physikalisch - sie ist nur in abstrakter Mathematik möglich.
@Kamil: Außerdem scheint ein Zeitintervall von Null Länge ein Widerspruch zu QM zu sein, wo Energiegrößen nicht unendlich und kontinuierlich auf Null zurückgehen können, aber die Größen von Entfernung und Zeit, die sie erzeugen - können. Sie haben die Planck-Zeit nicht ohne Grund.
@GeorgeRobinson Sie haben Newton-Mechanik getaggt , also lassen Sie bitte die Quantenmechanik weg. Newton (neben Leibniz) hat die Infinitesimalrechnung (abstrakte Mathematik!) entwickelt, um genau solche Probleme in der Newtonschen Mechanik zu lösen .

Ein Objekt mit einem Massenmittelpunkt, das einen Punkt auf einer Kreisbahn mit einem Radius umkreist R Positionsvektor hat

X ( T ) = R ( cos θ ( T ) , Sünde θ ( T ) )
und muss daher Nettokraft erfahren
F N e T = M R θ ˙ 2 ( cos θ , Sünde θ ) + M R θ ¨ ( Sünde θ , cos θ )
was eine Größenordnung hat
| F N e T | = M R θ ˙ 4 + θ ¨ 2

Damit die Größe der Kraft konstant ist, müssen wir haben

F F ˙ = 0
ω ˙ ( 2 ω 3 + ω ¨ ) = 0
Wo ω = θ ˙ ist die Winkelgeschwindigkeit. Es gibt zwei Lösungen: ω ˙ = 0 Und 2 ω 3 + ω ¨ = 0 . Die zweite Lösung funktioniert nicht, weil if ω > 0 Dann ω ¨ < 0 , aber das würde bedeuten, dass das Objekt nicht mit der gleichen Geschwindigkeit aus der halbkreisförmigen Bahn herauskommen könnte, mit der es begonnen hat. Das bedeutet, dass das Objekt den Halbkreis mit konstanter Geschwindigkeit durchlaufen muss ω ˙ = 0 .

Wenn wir uns die Gleichung für die Nettokraft ansehen, sehen wir, dass wenn θ ¨ = 0 , zeigt die Kraft immer zum Kreismittelpunkt. Und schließlich, wenn sich das Objekt nicht drehen soll, während es umkreist, muss die Kraft auch auf den Massenmittelpunkt des Objekts zeigen. Wenn sich das Objekt also mit konstanter Geschwindigkeit bewegt, ist Ihr Diagramm 1 die einzig richtige Antwort.

Wie bringen Sie Ihre Antwort mit dem Beweis unter imgur.com/LELihq9 in Einklang ? In Abb. 1 ist das anfängliche F1 senkrecht zu V0, sodass es nur dazu führen kann, dass senkrechte Geschwindigkeitskomponenten zu V0 hinzugefügt werden. Gemäß dem Beweis kann die MAGNITUDE der Vektorsumme von V0 + einem anderen Geschwindigkeitsvektor nicht gleich der Größe von V0 sein, wenn der Winkel zwischen den Summanden 90 Grad beträgt. Wenn Sie sehen möchten, was passiert, wenn ein weiterer Geschwindigkeitsvektor bei 90 Grad zu V0 hinzugefügt wird, dann sehen Sie sich dieses andere, wenn auch verwandte Szenario an: imgur.com/9dwtEPu
Das Problem bei diesem Beweis ist, dass er keine unendlich kleinen Geschwindigkeitsänderungen verwendet. Wenn wir wissen wollen, ob v hat zu jeder Zeit eine konstante Größe, von der wir die Ableitung nehmen müssen | v | = v v . Die Ableitung davon ist D | v | D T = A v | v | , die Drehzahl bleibt also nur dann konstant, wenn A v = 0 . Auch hier fügen wir keine Vektoren senkrecht zu hinzu v , addieren wir infinitesimale Vektoren senkrecht zu v . Es gibt kein
Ich dachte, dass die Regeln der Vektoraddition nicht von der Größe ihrer Summanden beeinflusst werden. Wie auch immer, wie kann das Hinzufügen von unendlich kleinen Vektoren, die PERPENDICULAR zu V0 sind, es auf (-V0) verringern (... und schließlich sogar umkehren)?
Weil Sie in jedem Moment ein neues Infinitesimal hinzufügen. Indem Sie zu jedem Zeitpunkt viele unendlich kleine Zahlen addieren, können Sie über einen bestimmten Zeitraum jede gewünschte Änderung erzielen. Sie müssen es so betrachten, weil Sie eine sich ständig ändernde Kraft haben und daher unendlich kleine Zeitänderungen berücksichtigen müssen.
Betrachten Sie zwei Geschwindigkeitsvektoren v A , B mit gleichem Betrag und Winkel θ zwischen ihnen. Der Winkel Δ v = v B v A macht mit v A wird bestimmt durch Δ v v A = v B v A | v A | 2 = | v A | 2 ( cos θ 1 ) . Da wir Geschwindigkeitsvektoren betrachten, die auf der Kreisbahn, die wir haben, sehr nahe beieinander liegen θ 0 und so cos θ 1 0 Und Δ v v A 0 .
...aber ist das realistisch? Gehen die Zeitintervalle zwischen Vektoradditionen wirklich unter die Planck-Zeit und nähern sich Null? Aus dieser Analyse entnehme ich, dass die Verwendung selbst lächerlich kurzer Intervalle wie der Planck-Zeit zu einem anderen Ergebnis führt als die Annäherung an Zeitintervalle der Länge Null. Denken Sie daran, ich bin an einer Antwort interessiert, die die Realität genau darstellt ... nicht an abstrakter Mathematik. PS Ist die ursprüngliche Frage wirklich so schlecht, dass sie keine Upvotes verdient?
Dies ist die grundlegende, zugrunde liegende Mathematik der klassischen Physik, die seit der Zeit von Newton verwendet wird. Wenn Sie glauben, dass die Newtonschen Gesetze die Realität korrekt beschreiben, glauben Sie, dass der Kalkül die Realität genau beschreibt. Anders ausgedrückt: Wenn Sie eine Beschleunigungsfunktion haben, die sich zeitlich konstant ändert, versuchen Sie, die Bewegung nur mit diskreten Zeitschritten zu berechnen Δ T wird immer einen Fehler haben, weil es ignoriert, wie sich die Beschleunigung zwischendurch geändert hat T Und T + Δ T .
Ich habe darüber nachgedacht und es scheint, dass Bewegung physikalisch nicht in einem Nullzeitintervall (einem Augenblick) stattfinden kann. Das ist nur in der abstrakten Mathematik möglich. IOW, in der Physik: Keine Zeit = keine Bewegung. Es scheint auch ein Widerspruch zur QM zu sein, wo Energiegrößen nicht unendlich und kontinuierlich auf Null zurückgehen können, aber die Größen von Entfernung und Zeit, die sie erzeugen, können.
Ist eine Bewegung in 0,1 s möglich? Ist es in 0,0001s möglich? Wie wäre es mit 0,000000000000001s? Oder 0,000000000000000000000000000000000000000001s? Stellen Sie die Hypothese auf, dass es eine kleinstmögliche Zeit gibt, nach der eine Änderung nicht möglich ist? Wenn ja, wo sind Ihre Beweise für diese Hypothese? In 600 Jahren Physik hat noch niemand einen physikalischen Beweis für die kleinste Zeit gefunden. Die gesamte klassische und moderne Physik baut auf der Vorstellung auf, Änderungen über willkürlich kleine Zeitintervalle zu berücksichtigen, und das ist die Idee, über die ich hier spreche. Niemand behauptet, dass Bewegung in kürzester Zeit passiert.
Solange das betrachtete Zeitintervall größer als Null ist, habe ich keine Einwände, obwohl die Planck-Zeit eine gewisse Begrenzung der Dauer darstellen KÖNNTE. Was ich wirklich ablehne, sind die Regeln der Vektoraddition, die von der Größe ihrer Summanden beeinflusst werden.
Niemand hat gesagt, dass sich die Regeln der Vektoraddition ändern, wenn sich die Größe ändert. Wenn Sie den Geschwindigkeitsvektor eines Objekts nehmen, das sich gleichzeitig im Kreis bewegt T und dann zur zeit T + Δ T dann der Beschleunigungsvektor A = ( v ( T + Δ T ) v ( T ) ) / Δ T ist ein Vektor, der sehr, sehr nahe an der Senkrechten steht v Wenn Δ T ist sehr, sehr klein. Je kleiner du machst Δ T , je näher A v ( T ) ist auf Null. Aus diesem Grund ist es sinnvoll zu sagen, dass die Beschleunigung zu jedem Zeitpunkt senkrecht zur Geschwindigkeit steht.

Um die Richtung des Fahrzeugs ohne orbitale Unterstützung umzukehren, wäre der treibstoffeffizienteste Weg, Triebwerke genau entgegengesetzt zur Fahrtrichtung abzufeuern, bis das Fahrzeug vollständig zum Stillstand kommt und sich dann zurückbewegt. Die gezeigten Diagramme würden das Fahrzeug drehen, aber seinen Kurs nicht effizient umkehren. Diagramm könnte man seine Grobheit umkehren, wenn die Triebwerke bei t3 kontinuierlich gezündet würden, bis das Fahrzeug vollständig zum Stillstand gekommen wäre und dann die gewünschte entgegengesetzte Geschwindigkeit erreicht hätte. Das bloße Drehen eines Projektils wird seine Grobheit nicht umkehren. Um ein Fahrzeug effizient zu drehen, benötigen Sie nur eine außermittige Verbrennung, um es zu drehen, und dann eine gleiche und entgegengesetzte Verbrennung, um die Drehung an der gewünschten Stelle zu stoppen.

Vielen Dank für die Antwort, aber die Raketentreibstoffeffizienz ist für dieses Problem irrelevant. Die halbkreisförmige Bahn ist obligatorisch.